Solve $a^3 + b^3 + c^3 = 6abc$












5












$begingroup$


Find solutions for $a^3 + b^3 + c^3 = 6abc$ in $mathbb{N}$, such that $gcd(a,b,c) = 1$, except for $(1,2,3)$ and its permutations.





Using trial and error I found out that if $a,b,c$ are solution of the equation, then they are in arithmetic progression. I've managed to prove that conjecture, assuming that $c>b>a$ and let $k$ be their common difference in the arithmetic progression. Then WLOG we have:



$$b = c-k quad quad a = c-2k$$



Now the equation looks like:



$$(c-2k)^3 + (c-k)^3 + c^3 = 6(c-2)(c-1)c$$



After expanding we have:



$$c^3 - 6kc^2 + 12ck^2 - 8k^3 + c^3 - 3kc^2 + 3ck^2 -k^3 + c^3 = 6c^3 - 18kc^2 + 12ck^2$$
$$3c^3 - 9kc^2 + 15ck^2 - 9k^3 = 6c^3 - 18kc^2 + 12ck^2$$
$$c^3 - 3kc^2 + 5ck^2 - 3k^3 = 2c^3 - 6kc^2 + 4ck^2$$
$$-c^3 + 3kc^2 + ck^2 - 3k^3 = 0$$



Now it's easy to see that if $k=c$, then the LHS will be zero, so one of the zeroes of the polynomial is $c_1 = k$, now factorizing we have:



$$(c-k)(3a^2 + 2ax - x^2) = 0$$
$$(c-k)(c+k)(c-3k) = 0$$



Now we have three distinct cases:



Case 1: $c = k$



This implies that $b = 0$ and $a = -k$. But because $k in mathbb{N}$, both $a,b notin mathbb{N}$, violating the initial conditions.



Case 2: $c = -k$



Obviously the initial condition is already violated, becasue $k in mathbb{N}$, so from the relation $c notin mathbb{N}$



Case 2: $c = 3k$



This implies that $b = 2k$ and $a = k$. Now we have one 3-tuple $(3k,2k,k)$ and it's permutation as solution, where $k in mathbb{N}$. But it's easy to note that $k$ is a common factor for $a,b,c$ so we have:



$$gcd(a,b,c) = k$$



But because we want $gcd(a,b,c) = 1$, this implies that $k=1$, which means we have only one solution for $a^3 + b^3 + c^3 = 6abc$ in $mathbb{N}$, such that $gcd(a,b,c) = 1$ and it $(1,2,3)$, solution that is already given.



Now my question is what I'm missing. Is there really no other solutions such that $gcd(a,b,c) = 1$? Or maybe there is a different way to obtain solution except for my method using arithmetic progression?










share|cite|improve this question











$endgroup$












  • $begingroup$
    saying that $a,b,c$ is an arithmetic progression satisfies the conditions is not correct, since $1,4,7$ doesn't satisfy the equation. What you mean is that ${ a, b, c } = {k, 2k, 3k } $, which agrees with your cases.
    $endgroup$
    – Calvin Lin
    Sep 25 '13 at 0:50










  • $begingroup$
    My bad. I didn't mean that. I meaned if $a,b,c$ are solution then it implies $a,b,c$ are in arithmetic progression, but it doesn't mean that every arithmetic progression will provide a solution. And what are you thoughts is there another solution satisfying the initial condition. Is there any other way to obtain it? Or there is just one?
    $endgroup$
    – Stefan4024
    Sep 25 '13 at 0:59










  • $begingroup$
    If the arithmetic progression condition is correct, then $(1,2,3)$ is the only primitive solution.
    $endgroup$
    – Daniel Fischer
    Sep 25 '13 at 1:34










  • $begingroup$
    I know that, but is there another way to generate (a,b,c) as integer solution except for arithemtic progression.
    $endgroup$
    – Stefan4024
    Sep 25 '13 at 1:35










  • $begingroup$
    @DanielFischer Is there any way to prove that at least one $a,b,c$ must be divisible by 3? If that's proven then it's easy to prove that (1,2,3) is the only primitive solution.
    $endgroup$
    – Stefan4024
    Sep 25 '13 at 2:13
















5












$begingroup$


Find solutions for $a^3 + b^3 + c^3 = 6abc$ in $mathbb{N}$, such that $gcd(a,b,c) = 1$, except for $(1,2,3)$ and its permutations.





Using trial and error I found out that if $a,b,c$ are solution of the equation, then they are in arithmetic progression. I've managed to prove that conjecture, assuming that $c>b>a$ and let $k$ be their common difference in the arithmetic progression. Then WLOG we have:



$$b = c-k quad quad a = c-2k$$



Now the equation looks like:



$$(c-2k)^3 + (c-k)^3 + c^3 = 6(c-2)(c-1)c$$



After expanding we have:



$$c^3 - 6kc^2 + 12ck^2 - 8k^3 + c^3 - 3kc^2 + 3ck^2 -k^3 + c^3 = 6c^3 - 18kc^2 + 12ck^2$$
$$3c^3 - 9kc^2 + 15ck^2 - 9k^3 = 6c^3 - 18kc^2 + 12ck^2$$
$$c^3 - 3kc^2 + 5ck^2 - 3k^3 = 2c^3 - 6kc^2 + 4ck^2$$
$$-c^3 + 3kc^2 + ck^2 - 3k^3 = 0$$



Now it's easy to see that if $k=c$, then the LHS will be zero, so one of the zeroes of the polynomial is $c_1 = k$, now factorizing we have:



$$(c-k)(3a^2 + 2ax - x^2) = 0$$
$$(c-k)(c+k)(c-3k) = 0$$



Now we have three distinct cases:



Case 1: $c = k$



This implies that $b = 0$ and $a = -k$. But because $k in mathbb{N}$, both $a,b notin mathbb{N}$, violating the initial conditions.



Case 2: $c = -k$



Obviously the initial condition is already violated, becasue $k in mathbb{N}$, so from the relation $c notin mathbb{N}$



Case 2: $c = 3k$



This implies that $b = 2k$ and $a = k$. Now we have one 3-tuple $(3k,2k,k)$ and it's permutation as solution, where $k in mathbb{N}$. But it's easy to note that $k$ is a common factor for $a,b,c$ so we have:



$$gcd(a,b,c) = k$$



But because we want $gcd(a,b,c) = 1$, this implies that $k=1$, which means we have only one solution for $a^3 + b^3 + c^3 = 6abc$ in $mathbb{N}$, such that $gcd(a,b,c) = 1$ and it $(1,2,3)$, solution that is already given.



Now my question is what I'm missing. Is there really no other solutions such that $gcd(a,b,c) = 1$? Or maybe there is a different way to obtain solution except for my method using arithmetic progression?










share|cite|improve this question











$endgroup$












  • $begingroup$
    saying that $a,b,c$ is an arithmetic progression satisfies the conditions is not correct, since $1,4,7$ doesn't satisfy the equation. What you mean is that ${ a, b, c } = {k, 2k, 3k } $, which agrees with your cases.
    $endgroup$
    – Calvin Lin
    Sep 25 '13 at 0:50










  • $begingroup$
    My bad. I didn't mean that. I meaned if $a,b,c$ are solution then it implies $a,b,c$ are in arithmetic progression, but it doesn't mean that every arithmetic progression will provide a solution. And what are you thoughts is there another solution satisfying the initial condition. Is there any other way to obtain it? Or there is just one?
    $endgroup$
    – Stefan4024
    Sep 25 '13 at 0:59










  • $begingroup$
    If the arithmetic progression condition is correct, then $(1,2,3)$ is the only primitive solution.
    $endgroup$
    – Daniel Fischer
    Sep 25 '13 at 1:34










  • $begingroup$
    I know that, but is there another way to generate (a,b,c) as integer solution except for arithemtic progression.
    $endgroup$
    – Stefan4024
    Sep 25 '13 at 1:35










  • $begingroup$
    @DanielFischer Is there any way to prove that at least one $a,b,c$ must be divisible by 3? If that's proven then it's easy to prove that (1,2,3) is the only primitive solution.
    $endgroup$
    – Stefan4024
    Sep 25 '13 at 2:13














5












5








5


1



$begingroup$


Find solutions for $a^3 + b^3 + c^3 = 6abc$ in $mathbb{N}$, such that $gcd(a,b,c) = 1$, except for $(1,2,3)$ and its permutations.





Using trial and error I found out that if $a,b,c$ are solution of the equation, then they are in arithmetic progression. I've managed to prove that conjecture, assuming that $c>b>a$ and let $k$ be their common difference in the arithmetic progression. Then WLOG we have:



$$b = c-k quad quad a = c-2k$$



Now the equation looks like:



$$(c-2k)^3 + (c-k)^3 + c^3 = 6(c-2)(c-1)c$$



After expanding we have:



$$c^3 - 6kc^2 + 12ck^2 - 8k^3 + c^3 - 3kc^2 + 3ck^2 -k^3 + c^3 = 6c^3 - 18kc^2 + 12ck^2$$
$$3c^3 - 9kc^2 + 15ck^2 - 9k^3 = 6c^3 - 18kc^2 + 12ck^2$$
$$c^3 - 3kc^2 + 5ck^2 - 3k^3 = 2c^3 - 6kc^2 + 4ck^2$$
$$-c^3 + 3kc^2 + ck^2 - 3k^3 = 0$$



Now it's easy to see that if $k=c$, then the LHS will be zero, so one of the zeroes of the polynomial is $c_1 = k$, now factorizing we have:



$$(c-k)(3a^2 + 2ax - x^2) = 0$$
$$(c-k)(c+k)(c-3k) = 0$$



Now we have three distinct cases:



Case 1: $c = k$



This implies that $b = 0$ and $a = -k$. But because $k in mathbb{N}$, both $a,b notin mathbb{N}$, violating the initial conditions.



Case 2: $c = -k$



Obviously the initial condition is already violated, becasue $k in mathbb{N}$, so from the relation $c notin mathbb{N}$



Case 2: $c = 3k$



This implies that $b = 2k$ and $a = k$. Now we have one 3-tuple $(3k,2k,k)$ and it's permutation as solution, where $k in mathbb{N}$. But it's easy to note that $k$ is a common factor for $a,b,c$ so we have:



$$gcd(a,b,c) = k$$



But because we want $gcd(a,b,c) = 1$, this implies that $k=1$, which means we have only one solution for $a^3 + b^3 + c^3 = 6abc$ in $mathbb{N}$, such that $gcd(a,b,c) = 1$ and it $(1,2,3)$, solution that is already given.



Now my question is what I'm missing. Is there really no other solutions such that $gcd(a,b,c) = 1$? Or maybe there is a different way to obtain solution except for my method using arithmetic progression?










share|cite|improve this question











$endgroup$




Find solutions for $a^3 + b^3 + c^3 = 6abc$ in $mathbb{N}$, such that $gcd(a,b,c) = 1$, except for $(1,2,3)$ and its permutations.





Using trial and error I found out that if $a,b,c$ are solution of the equation, then they are in arithmetic progression. I've managed to prove that conjecture, assuming that $c>b>a$ and let $k$ be their common difference in the arithmetic progression. Then WLOG we have:



$$b = c-k quad quad a = c-2k$$



Now the equation looks like:



$$(c-2k)^3 + (c-k)^3 + c^3 = 6(c-2)(c-1)c$$



After expanding we have:



$$c^3 - 6kc^2 + 12ck^2 - 8k^3 + c^3 - 3kc^2 + 3ck^2 -k^3 + c^3 = 6c^3 - 18kc^2 + 12ck^2$$
$$3c^3 - 9kc^2 + 15ck^2 - 9k^3 = 6c^3 - 18kc^2 + 12ck^2$$
$$c^3 - 3kc^2 + 5ck^2 - 3k^3 = 2c^3 - 6kc^2 + 4ck^2$$
$$-c^3 + 3kc^2 + ck^2 - 3k^3 = 0$$



Now it's easy to see that if $k=c$, then the LHS will be zero, so one of the zeroes of the polynomial is $c_1 = k$, now factorizing we have:



$$(c-k)(3a^2 + 2ax - x^2) = 0$$
$$(c-k)(c+k)(c-3k) = 0$$



Now we have three distinct cases:



Case 1: $c = k$



This implies that $b = 0$ and $a = -k$. But because $k in mathbb{N}$, both $a,b notin mathbb{N}$, violating the initial conditions.



Case 2: $c = -k$



Obviously the initial condition is already violated, becasue $k in mathbb{N}$, so from the relation $c notin mathbb{N}$



Case 2: $c = 3k$



This implies that $b = 2k$ and $a = k$. Now we have one 3-tuple $(3k,2k,k)$ and it's permutation as solution, where $k in mathbb{N}$. But it's easy to note that $k$ is a common factor for $a,b,c$ so we have:



$$gcd(a,b,c) = k$$



But because we want $gcd(a,b,c) = 1$, this implies that $k=1$, which means we have only one solution for $a^3 + b^3 + c^3 = 6abc$ in $mathbb{N}$, such that $gcd(a,b,c) = 1$ and it $(1,2,3)$, solution that is already given.



Now my question is what I'm missing. Is there really no other solutions such that $gcd(a,b,c) = 1$? Or maybe there is a different way to obtain solution except for my method using arithmetic progression?







sequences-and-series polynomials factoring divisibility






share|cite|improve this question















share|cite|improve this question













share|cite|improve this question




share|cite|improve this question








edited Sep 25 '13 at 1:00







Stefan4024

















asked Sep 25 '13 at 0:21









Stefan4024Stefan4024

30.6k63579




30.6k63579












  • $begingroup$
    saying that $a,b,c$ is an arithmetic progression satisfies the conditions is not correct, since $1,4,7$ doesn't satisfy the equation. What you mean is that ${ a, b, c } = {k, 2k, 3k } $, which agrees with your cases.
    $endgroup$
    – Calvin Lin
    Sep 25 '13 at 0:50










  • $begingroup$
    My bad. I didn't mean that. I meaned if $a,b,c$ are solution then it implies $a,b,c$ are in arithmetic progression, but it doesn't mean that every arithmetic progression will provide a solution. And what are you thoughts is there another solution satisfying the initial condition. Is there any other way to obtain it? Or there is just one?
    $endgroup$
    – Stefan4024
    Sep 25 '13 at 0:59










  • $begingroup$
    If the arithmetic progression condition is correct, then $(1,2,3)$ is the only primitive solution.
    $endgroup$
    – Daniel Fischer
    Sep 25 '13 at 1:34










  • $begingroup$
    I know that, but is there another way to generate (a,b,c) as integer solution except for arithemtic progression.
    $endgroup$
    – Stefan4024
    Sep 25 '13 at 1:35










  • $begingroup$
    @DanielFischer Is there any way to prove that at least one $a,b,c$ must be divisible by 3? If that's proven then it's easy to prove that (1,2,3) is the only primitive solution.
    $endgroup$
    – Stefan4024
    Sep 25 '13 at 2:13


















  • $begingroup$
    saying that $a,b,c$ is an arithmetic progression satisfies the conditions is not correct, since $1,4,7$ doesn't satisfy the equation. What you mean is that ${ a, b, c } = {k, 2k, 3k } $, which agrees with your cases.
    $endgroup$
    – Calvin Lin
    Sep 25 '13 at 0:50










  • $begingroup$
    My bad. I didn't mean that. I meaned if $a,b,c$ are solution then it implies $a,b,c$ are in arithmetic progression, but it doesn't mean that every arithmetic progression will provide a solution. And what are you thoughts is there another solution satisfying the initial condition. Is there any other way to obtain it? Or there is just one?
    $endgroup$
    – Stefan4024
    Sep 25 '13 at 0:59










  • $begingroup$
    If the arithmetic progression condition is correct, then $(1,2,3)$ is the only primitive solution.
    $endgroup$
    – Daniel Fischer
    Sep 25 '13 at 1:34










  • $begingroup$
    I know that, but is there another way to generate (a,b,c) as integer solution except for arithemtic progression.
    $endgroup$
    – Stefan4024
    Sep 25 '13 at 1:35










  • $begingroup$
    @DanielFischer Is there any way to prove that at least one $a,b,c$ must be divisible by 3? If that's proven then it's easy to prove that (1,2,3) is the only primitive solution.
    $endgroup$
    – Stefan4024
    Sep 25 '13 at 2:13
















$begingroup$
saying that $a,b,c$ is an arithmetic progression satisfies the conditions is not correct, since $1,4,7$ doesn't satisfy the equation. What you mean is that ${ a, b, c } = {k, 2k, 3k } $, which agrees with your cases.
$endgroup$
– Calvin Lin
Sep 25 '13 at 0:50




$begingroup$
saying that $a,b,c$ is an arithmetic progression satisfies the conditions is not correct, since $1,4,7$ doesn't satisfy the equation. What you mean is that ${ a, b, c } = {k, 2k, 3k } $, which agrees with your cases.
$endgroup$
– Calvin Lin
Sep 25 '13 at 0:50












$begingroup$
My bad. I didn't mean that. I meaned if $a,b,c$ are solution then it implies $a,b,c$ are in arithmetic progression, but it doesn't mean that every arithmetic progression will provide a solution. And what are you thoughts is there another solution satisfying the initial condition. Is there any other way to obtain it? Or there is just one?
$endgroup$
– Stefan4024
Sep 25 '13 at 0:59




$begingroup$
My bad. I didn't mean that. I meaned if $a,b,c$ are solution then it implies $a,b,c$ are in arithmetic progression, but it doesn't mean that every arithmetic progression will provide a solution. And what are you thoughts is there another solution satisfying the initial condition. Is there any other way to obtain it? Or there is just one?
$endgroup$
– Stefan4024
Sep 25 '13 at 0:59












$begingroup$
If the arithmetic progression condition is correct, then $(1,2,3)$ is the only primitive solution.
$endgroup$
– Daniel Fischer
Sep 25 '13 at 1:34




$begingroup$
If the arithmetic progression condition is correct, then $(1,2,3)$ is the only primitive solution.
$endgroup$
– Daniel Fischer
Sep 25 '13 at 1:34












$begingroup$
I know that, but is there another way to generate (a,b,c) as integer solution except for arithemtic progression.
$endgroup$
– Stefan4024
Sep 25 '13 at 1:35




$begingroup$
I know that, but is there another way to generate (a,b,c) as integer solution except for arithemtic progression.
$endgroup$
– Stefan4024
Sep 25 '13 at 1:35












$begingroup$
@DanielFischer Is there any way to prove that at least one $a,b,c$ must be divisible by 3? If that's proven then it's easy to prove that (1,2,3) is the only primitive solution.
$endgroup$
– Stefan4024
Sep 25 '13 at 2:13




$begingroup$
@DanielFischer Is there any way to prove that at least one $a,b,c$ must be divisible by 3? If that's proven then it's easy to prove that (1,2,3) is the only primitive solution.
$endgroup$
– Stefan4024
Sep 25 '13 at 2:13










1 Answer
1






active

oldest

votes


















0












$begingroup$

It's easy to prove that at least one of the variables needs to be an even number. We know that:



$$6|(n-1)n(n+1)$$



Because in three consecutive numbers, one is divisible with three and at least one is divisible with 2. So we have:



$$6|n^3 - n$$
$$n^3 = n pmod 6$$



Now we have:



$$a^3 + b^3 + c^3 equiv a + b + c equiv 0 pmod 6$$



Beacuse the modulo is an even number that means that the sum $a+b+c$ is an even number also. We know that the sum of 3 odd numbers will be odd number, so it's impossible $a,b,c$ to be odd number, because there won't be solution. So it means that at least one of the variables is an even number.



WLOG we can set $b=2k$. Now we can continue:



$$b-2k = 0$$



Now we can multiply both sides with $b(b+2k)$. Note that won't give another solution, because it'll imply that b is $0$ or a negative number, which violate the condition. Now we have:



$$b(b+2k)(b-2k) = 0$$
$$b(b^2 - 4k^2) = 0$$
$$b^3 - 4bk^2 = 0$$
$$3b^3 - 12bk^2 = 0$$
$$6b^3 - 6bk^2 = 3b^3 + 6bk^2 = 0$$
$$6b(b^2 - k^2) = (b^2 - 3kb^2 + 3bk^2 - k^3) + b^3 + (b^3 + 3kb^2 + 3bk^2 + k^3) = 0$$
$$6b(b-k)(b+k) = (b-k)^3 + b^3 + (b+k)^3$$



Now if we substitute WLOG:



$$b+k=c quad quad b-k=a$$



$$6abc = a^3 + b^3 + c^3$$



Because $b,k in mathbb{N}$ it means that also $a,c in mathbb{N}$. So this proves that for any $b=2k$, there are integer solutions, such $a=k$ and $c=3k$.



But because $k$ is a factor of all of them it's easy to see that:



$$gcd(a,b,c) = gcd(k,2k,3k) = k$$



Because we want $gcd(a,b,c) = 1$, that implies that $k=1$ and that the only primitive solution of this equation is $(1,2,3)$ and its permutation.






share|cite|improve this answer









$endgroup$









  • 1




    $begingroup$
    This doesn't explain how to find all triples of solutions. Note that right now, you have may merely used algebraic manipulations, without any serious number theory. There is no reason why the answers must appear in an arithmetic progression. For the initial argument, you can shorten it to the cube of an odd number is odd, and the sum of 3 odd numbers is odd.
    $endgroup$
    – Calvin Lin
    Sep 25 '13 at 13:49










  • $begingroup$
    Actually that's my problem, I know that any arithmetic progression of the type {k,2k,3k} would provide a solution, but I can't prove that that's the only way to obtain solutions for this problem.
    $endgroup$
    – Stefan4024
    Sep 25 '13 at 17:29











Your Answer





StackExchange.ifUsing("editor", function () {
return StackExchange.using("mathjaxEditing", function () {
StackExchange.MarkdownEditor.creationCallbacks.add(function (editor, postfix) {
StackExchange.mathjaxEditing.prepareWmdForMathJax(editor, postfix, [["$", "$"], ["\\(","\\)"]]);
});
});
}, "mathjax-editing");

StackExchange.ready(function() {
var channelOptions = {
tags: "".split(" "),
id: "69"
};
initTagRenderer("".split(" "), "".split(" "), channelOptions);

StackExchange.using("externalEditor", function() {
// Have to fire editor after snippets, if snippets enabled
if (StackExchange.settings.snippets.snippetsEnabled) {
StackExchange.using("snippets", function() {
createEditor();
});
}
else {
createEditor();
}
});

function createEditor() {
StackExchange.prepareEditor({
heartbeatType: 'answer',
autoActivateHeartbeat: false,
convertImagesToLinks: true,
noModals: true,
showLowRepImageUploadWarning: true,
reputationToPostImages: 10,
bindNavPrevention: true,
postfix: "",
imageUploader: {
brandingHtml: "Powered by u003ca class="icon-imgur-white" href="https://imgur.com/"u003eu003c/au003e",
contentPolicyHtml: "User contributions licensed under u003ca href="https://creativecommons.org/licenses/by-sa/3.0/"u003ecc by-sa 3.0 with attribution requiredu003c/au003e u003ca href="https://stackoverflow.com/legal/content-policy"u003e(content policy)u003c/au003e",
allowUrls: true
},
noCode: true, onDemand: true,
discardSelector: ".discard-answer"
,immediatelyShowMarkdownHelp:true
});


}
});














draft saved

draft discarded


















StackExchange.ready(
function () {
StackExchange.openid.initPostLogin('.new-post-login', 'https%3a%2f%2fmath.stackexchange.com%2fquestions%2f504170%2fsolve-a3-b3-c3-6abc%23new-answer', 'question_page');
}
);

Post as a guest















Required, but never shown

























1 Answer
1






active

oldest

votes








1 Answer
1






active

oldest

votes









active

oldest

votes






active

oldest

votes









0












$begingroup$

It's easy to prove that at least one of the variables needs to be an even number. We know that:



$$6|(n-1)n(n+1)$$



Because in three consecutive numbers, one is divisible with three and at least one is divisible with 2. So we have:



$$6|n^3 - n$$
$$n^3 = n pmod 6$$



Now we have:



$$a^3 + b^3 + c^3 equiv a + b + c equiv 0 pmod 6$$



Beacuse the modulo is an even number that means that the sum $a+b+c$ is an even number also. We know that the sum of 3 odd numbers will be odd number, so it's impossible $a,b,c$ to be odd number, because there won't be solution. So it means that at least one of the variables is an even number.



WLOG we can set $b=2k$. Now we can continue:



$$b-2k = 0$$



Now we can multiply both sides with $b(b+2k)$. Note that won't give another solution, because it'll imply that b is $0$ or a negative number, which violate the condition. Now we have:



$$b(b+2k)(b-2k) = 0$$
$$b(b^2 - 4k^2) = 0$$
$$b^3 - 4bk^2 = 0$$
$$3b^3 - 12bk^2 = 0$$
$$6b^3 - 6bk^2 = 3b^3 + 6bk^2 = 0$$
$$6b(b^2 - k^2) = (b^2 - 3kb^2 + 3bk^2 - k^3) + b^3 + (b^3 + 3kb^2 + 3bk^2 + k^3) = 0$$
$$6b(b-k)(b+k) = (b-k)^3 + b^3 + (b+k)^3$$



Now if we substitute WLOG:



$$b+k=c quad quad b-k=a$$



$$6abc = a^3 + b^3 + c^3$$



Because $b,k in mathbb{N}$ it means that also $a,c in mathbb{N}$. So this proves that for any $b=2k$, there are integer solutions, such $a=k$ and $c=3k$.



But because $k$ is a factor of all of them it's easy to see that:



$$gcd(a,b,c) = gcd(k,2k,3k) = k$$



Because we want $gcd(a,b,c) = 1$, that implies that $k=1$ and that the only primitive solution of this equation is $(1,2,3)$ and its permutation.






share|cite|improve this answer









$endgroup$









  • 1




    $begingroup$
    This doesn't explain how to find all triples of solutions. Note that right now, you have may merely used algebraic manipulations, without any serious number theory. There is no reason why the answers must appear in an arithmetic progression. For the initial argument, you can shorten it to the cube of an odd number is odd, and the sum of 3 odd numbers is odd.
    $endgroup$
    – Calvin Lin
    Sep 25 '13 at 13:49










  • $begingroup$
    Actually that's my problem, I know that any arithmetic progression of the type {k,2k,3k} would provide a solution, but I can't prove that that's the only way to obtain solutions for this problem.
    $endgroup$
    – Stefan4024
    Sep 25 '13 at 17:29
















0












$begingroup$

It's easy to prove that at least one of the variables needs to be an even number. We know that:



$$6|(n-1)n(n+1)$$



Because in three consecutive numbers, one is divisible with three and at least one is divisible with 2. So we have:



$$6|n^3 - n$$
$$n^3 = n pmod 6$$



Now we have:



$$a^3 + b^3 + c^3 equiv a + b + c equiv 0 pmod 6$$



Beacuse the modulo is an even number that means that the sum $a+b+c$ is an even number also. We know that the sum of 3 odd numbers will be odd number, so it's impossible $a,b,c$ to be odd number, because there won't be solution. So it means that at least one of the variables is an even number.



WLOG we can set $b=2k$. Now we can continue:



$$b-2k = 0$$



Now we can multiply both sides with $b(b+2k)$. Note that won't give another solution, because it'll imply that b is $0$ or a negative number, which violate the condition. Now we have:



$$b(b+2k)(b-2k) = 0$$
$$b(b^2 - 4k^2) = 0$$
$$b^3 - 4bk^2 = 0$$
$$3b^3 - 12bk^2 = 0$$
$$6b^3 - 6bk^2 = 3b^3 + 6bk^2 = 0$$
$$6b(b^2 - k^2) = (b^2 - 3kb^2 + 3bk^2 - k^3) + b^3 + (b^3 + 3kb^2 + 3bk^2 + k^3) = 0$$
$$6b(b-k)(b+k) = (b-k)^3 + b^3 + (b+k)^3$$



Now if we substitute WLOG:



$$b+k=c quad quad b-k=a$$



$$6abc = a^3 + b^3 + c^3$$



Because $b,k in mathbb{N}$ it means that also $a,c in mathbb{N}$. So this proves that for any $b=2k$, there are integer solutions, such $a=k$ and $c=3k$.



But because $k$ is a factor of all of them it's easy to see that:



$$gcd(a,b,c) = gcd(k,2k,3k) = k$$



Because we want $gcd(a,b,c) = 1$, that implies that $k=1$ and that the only primitive solution of this equation is $(1,2,3)$ and its permutation.






share|cite|improve this answer









$endgroup$









  • 1




    $begingroup$
    This doesn't explain how to find all triples of solutions. Note that right now, you have may merely used algebraic manipulations, without any serious number theory. There is no reason why the answers must appear in an arithmetic progression. For the initial argument, you can shorten it to the cube of an odd number is odd, and the sum of 3 odd numbers is odd.
    $endgroup$
    – Calvin Lin
    Sep 25 '13 at 13:49










  • $begingroup$
    Actually that's my problem, I know that any arithmetic progression of the type {k,2k,3k} would provide a solution, but I can't prove that that's the only way to obtain solutions for this problem.
    $endgroup$
    – Stefan4024
    Sep 25 '13 at 17:29














0












0








0





$begingroup$

It's easy to prove that at least one of the variables needs to be an even number. We know that:



$$6|(n-1)n(n+1)$$



Because in three consecutive numbers, one is divisible with three and at least one is divisible with 2. So we have:



$$6|n^3 - n$$
$$n^3 = n pmod 6$$



Now we have:



$$a^3 + b^3 + c^3 equiv a + b + c equiv 0 pmod 6$$



Beacuse the modulo is an even number that means that the sum $a+b+c$ is an even number also. We know that the sum of 3 odd numbers will be odd number, so it's impossible $a,b,c$ to be odd number, because there won't be solution. So it means that at least one of the variables is an even number.



WLOG we can set $b=2k$. Now we can continue:



$$b-2k = 0$$



Now we can multiply both sides with $b(b+2k)$. Note that won't give another solution, because it'll imply that b is $0$ or a negative number, which violate the condition. Now we have:



$$b(b+2k)(b-2k) = 0$$
$$b(b^2 - 4k^2) = 0$$
$$b^3 - 4bk^2 = 0$$
$$3b^3 - 12bk^2 = 0$$
$$6b^3 - 6bk^2 = 3b^3 + 6bk^2 = 0$$
$$6b(b^2 - k^2) = (b^2 - 3kb^2 + 3bk^2 - k^3) + b^3 + (b^3 + 3kb^2 + 3bk^2 + k^3) = 0$$
$$6b(b-k)(b+k) = (b-k)^3 + b^3 + (b+k)^3$$



Now if we substitute WLOG:



$$b+k=c quad quad b-k=a$$



$$6abc = a^3 + b^3 + c^3$$



Because $b,k in mathbb{N}$ it means that also $a,c in mathbb{N}$. So this proves that for any $b=2k$, there are integer solutions, such $a=k$ and $c=3k$.



But because $k$ is a factor of all of them it's easy to see that:



$$gcd(a,b,c) = gcd(k,2k,3k) = k$$



Because we want $gcd(a,b,c) = 1$, that implies that $k=1$ and that the only primitive solution of this equation is $(1,2,3)$ and its permutation.






share|cite|improve this answer









$endgroup$



It's easy to prove that at least one of the variables needs to be an even number. We know that:



$$6|(n-1)n(n+1)$$



Because in three consecutive numbers, one is divisible with three and at least one is divisible with 2. So we have:



$$6|n^3 - n$$
$$n^3 = n pmod 6$$



Now we have:



$$a^3 + b^3 + c^3 equiv a + b + c equiv 0 pmod 6$$



Beacuse the modulo is an even number that means that the sum $a+b+c$ is an even number also. We know that the sum of 3 odd numbers will be odd number, so it's impossible $a,b,c$ to be odd number, because there won't be solution. So it means that at least one of the variables is an even number.



WLOG we can set $b=2k$. Now we can continue:



$$b-2k = 0$$



Now we can multiply both sides with $b(b+2k)$. Note that won't give another solution, because it'll imply that b is $0$ or a negative number, which violate the condition. Now we have:



$$b(b+2k)(b-2k) = 0$$
$$b(b^2 - 4k^2) = 0$$
$$b^3 - 4bk^2 = 0$$
$$3b^3 - 12bk^2 = 0$$
$$6b^3 - 6bk^2 = 3b^3 + 6bk^2 = 0$$
$$6b(b^2 - k^2) = (b^2 - 3kb^2 + 3bk^2 - k^3) + b^3 + (b^3 + 3kb^2 + 3bk^2 + k^3) = 0$$
$$6b(b-k)(b+k) = (b-k)^3 + b^3 + (b+k)^3$$



Now if we substitute WLOG:



$$b+k=c quad quad b-k=a$$



$$6abc = a^3 + b^3 + c^3$$



Because $b,k in mathbb{N}$ it means that also $a,c in mathbb{N}$. So this proves that for any $b=2k$, there are integer solutions, such $a=k$ and $c=3k$.



But because $k$ is a factor of all of them it's easy to see that:



$$gcd(a,b,c) = gcd(k,2k,3k) = k$$



Because we want $gcd(a,b,c) = 1$, that implies that $k=1$ and that the only primitive solution of this equation is $(1,2,3)$ and its permutation.







share|cite|improve this answer












share|cite|improve this answer



share|cite|improve this answer










answered Sep 25 '13 at 8:38









Stefan4024Stefan4024

30.6k63579




30.6k63579








  • 1




    $begingroup$
    This doesn't explain how to find all triples of solutions. Note that right now, you have may merely used algebraic manipulations, without any serious number theory. There is no reason why the answers must appear in an arithmetic progression. For the initial argument, you can shorten it to the cube of an odd number is odd, and the sum of 3 odd numbers is odd.
    $endgroup$
    – Calvin Lin
    Sep 25 '13 at 13:49










  • $begingroup$
    Actually that's my problem, I know that any arithmetic progression of the type {k,2k,3k} would provide a solution, but I can't prove that that's the only way to obtain solutions for this problem.
    $endgroup$
    – Stefan4024
    Sep 25 '13 at 17:29














  • 1




    $begingroup$
    This doesn't explain how to find all triples of solutions. Note that right now, you have may merely used algebraic manipulations, without any serious number theory. There is no reason why the answers must appear in an arithmetic progression. For the initial argument, you can shorten it to the cube of an odd number is odd, and the sum of 3 odd numbers is odd.
    $endgroup$
    – Calvin Lin
    Sep 25 '13 at 13:49










  • $begingroup$
    Actually that's my problem, I know that any arithmetic progression of the type {k,2k,3k} would provide a solution, but I can't prove that that's the only way to obtain solutions for this problem.
    $endgroup$
    – Stefan4024
    Sep 25 '13 at 17:29








1




1




$begingroup$
This doesn't explain how to find all triples of solutions. Note that right now, you have may merely used algebraic manipulations, without any serious number theory. There is no reason why the answers must appear in an arithmetic progression. For the initial argument, you can shorten it to the cube of an odd number is odd, and the sum of 3 odd numbers is odd.
$endgroup$
– Calvin Lin
Sep 25 '13 at 13:49




$begingroup$
This doesn't explain how to find all triples of solutions. Note that right now, you have may merely used algebraic manipulations, without any serious number theory. There is no reason why the answers must appear in an arithmetic progression. For the initial argument, you can shorten it to the cube of an odd number is odd, and the sum of 3 odd numbers is odd.
$endgroup$
– Calvin Lin
Sep 25 '13 at 13:49












$begingroup$
Actually that's my problem, I know that any arithmetic progression of the type {k,2k,3k} would provide a solution, but I can't prove that that's the only way to obtain solutions for this problem.
$endgroup$
– Stefan4024
Sep 25 '13 at 17:29




$begingroup$
Actually that's my problem, I know that any arithmetic progression of the type {k,2k,3k} would provide a solution, but I can't prove that that's the only way to obtain solutions for this problem.
$endgroup$
– Stefan4024
Sep 25 '13 at 17:29


















draft saved

draft discarded




















































Thanks for contributing an answer to Mathematics Stack Exchange!


  • Please be sure to answer the question. Provide details and share your research!

But avoid



  • Asking for help, clarification, or responding to other answers.

  • Making statements based on opinion; back them up with references or personal experience.


Use MathJax to format equations. MathJax reference.


To learn more, see our tips on writing great answers.




draft saved


draft discarded














StackExchange.ready(
function () {
StackExchange.openid.initPostLogin('.new-post-login', 'https%3a%2f%2fmath.stackexchange.com%2fquestions%2f504170%2fsolve-a3-b3-c3-6abc%23new-answer', 'question_page');
}
);

Post as a guest















Required, but never shown





















































Required, but never shown














Required, but never shown












Required, but never shown







Required, but never shown

































Required, but never shown














Required, but never shown












Required, but never shown







Required, but never shown







Popular posts from this blog

Can a sorcerer learn a 5th-level spell early by creating spell slots using the Font of Magic feature?

ts Property 'filter' does not exist on type '{}'

Notepad++ export/extract a list of installed plugins